11.09.2015 Views

MATH 1105, PARTIAL EXAM 1 SOLUTIONS TO PRACTICE ...

MATH 1105, PARTIAL EXAM 1 SOLUTIONS TO PRACTICE ...

MATH 1105, PARTIAL EXAM 1 SOLUTIONS TO PRACTICE ...

SHOW MORE
SHOW LESS
  • No tags were found...

Create successful ePaper yourself

Turn your PDF publications into a flip-book with our unique Google optimized e-Paper software.

<strong>MATH</strong> <strong>1105</strong>, <strong>PARTIAL</strong> <strong>EXAM</strong> 1<br />

<strong>SOLUTIONS</strong> <strong>TO</strong> <strong>PRACTICE</strong> PROBLEMS<br />

JOHN GOODRICK<br />

Problem 1. For each of the matrices below, determine whether or not it is<br />

in row echelon form. If it is not, put it into row echelon form using a single<br />

elementary row operation.<br />

A =<br />

⎡<br />

⎣ 1 2 3 −1<br />

0 1 7 7<br />

0 1 3 0<br />

⎤<br />

⎦ B =<br />

⎡<br />

⎣ 0 0 1<br />

0 1 3<br />

1 −1 2<br />

⎤<br />

⎦ C =<br />

⎡<br />

⎣ 1 3 1<br />

0 1 7<br />

0 0 −4<br />

SOLUTION: The matrix A is not in row echelon form – the leading entry of<br />

row 3 is not to the right of the first nonzero entry of row 2. (See the definition<br />

on p. 57 of the textbook.) However, by subtracting row 2 from row 3, we get<br />

the following matrix, which is in row echelon form:<br />

⎡<br />

⎣ 1 2 3 −1<br />

0 1 7 7<br />

0 0 −4 −7<br />

The matrix B is also not in row echelon form, for the same reason as A is<br />

not. By swapping row 1 and row 3, we do get a matrix in row echelon form:<br />

B ∼<br />

⎡<br />

⎣ 1 −1 2<br />

0 1 3<br />

0 0 1<br />

The matrix C is in row echelon form. (It is not in reduced row echelon<br />

form, since the pivot −4 in row 3 is not equal to 1, but this wasn’t what I was<br />

asking about.) If this is unclear, you should review the definition on p. 57 of<br />

the textbook.<br />

Problem 2.<br />

(a) Find the general solution to the linear system<br />

⎤<br />

⎦<br />

⎤<br />

⎦<br />

x 1 + 2x 2 − x 3 + 2x 4 = 0<br />

3x 1 + 6x 2 − 3x 3 + 9x 4 = 0.<br />

SOLUTION: Put the system into an augmented matrix, then row reduce,<br />

just as described in section 1.4 of the textbook:<br />

1<br />

⎤<br />


2 JOHN GOODRICK<br />

[ ] [ ] [ ]<br />

1 2 −1 2 | 0 1 2 −1 2 | 0 1 2 −1 2 | 0<br />

∼<br />

∼<br />

3 6 −3 9 | 0 0 0 0 3 | 0 0 0 0 1 | 0<br />

[ ]<br />

1 2 −1 0 | 0<br />

∼<br />

.<br />

0 0 0 1 | 0<br />

From here, we can read off the general solution: x 1 = −2x 2 + x 3 − 2x 4 ,<br />

x 4 = 0, and since there are no pivots in columns 2 and 3, the variables x 2 and<br />

x 3 are free.<br />

(Don’t forget to say that x 2 and x 3 are free variables!)<br />

(b) Is there any pair of numbers b 1 and b 2 in R such that the system<br />

x 1 + 2x 2 − x 3 + 2x 4 = b 1<br />

3x 1 + 6x 2 − 3x 3 + 9x 4 = b 2<br />

has no solution for x 1 , x 2 , x 3 , and x 4 ? Explain why or why not.<br />

If you write out the augmented matrix for the system with b 1 and b 2 and<br />

row-reduce it like in part (a), you will get:<br />

[ ] [ ]<br />

1 2 −1 2 | b1 1 2 −1 2 | b1<br />

∼<br />

3 6 −3 9 | b 2 0 0 0 3 | b 2 − 3b 1<br />

[ ] [ ]<br />

1 2 −1 2 | b1 1 2 −1 0 | 3b1 − 2b 2<br />

∼<br />

b<br />

0 0 0 1 | 23<br />

∼<br />

3<br />

b<br />

− b 1 0 0 0 1 | 23<br />

.<br />

− b 1<br />

Notice that no matter what the numbers b 1 and b 2 are, the final matrix has<br />

pivots in columns 1 and 4, but no pivot in the rightmost column – therefore<br />

the solution is always consistent, and there is no pair of numbers b 1 and b 2<br />

for which the system has no solution.<br />

Problem 3. (a) Let<br />

⎧⎡<br />

⎨<br />

S = ⎣ 1 2<br />

⎩<br />

3<br />

⎤<br />

⎦ ,<br />

⎡<br />

⎣ 4 5<br />

6<br />

⎤<br />

⎦ ,<br />

⎡<br />

⎣ 7 8<br />

9<br />

⎤⎫<br />

⎬<br />

⎦<br />

⎭ .<br />

Is S linearly independent? Explain why or why not.<br />

SOLUTION: The best general way to do a problem like this is to put the<br />

vectors into a matrix A (as columns), then row-reduce:<br />

⎡<br />

A = ⎣ 1 4 7<br />

⎤ ⎡<br />

2 5 8 ⎦ ∼ ⎣ 1 4 7<br />

⎤ ⎡<br />

0 −3 −6 ⎦ ∼ ⎣ 1 4 7<br />

⎤<br />

0 −3 −6 ⎦ .<br />

3 6 9 0 −6 −12 0 0 0<br />

This last matrix is in row echelon form, but there is no pivot in the third<br />

column – therefore S is linearly dependent.


<strong>MATH</strong> <strong>1105</strong>, <strong>PARTIAL</strong> <strong>EXAM</strong> 1 <strong>SOLUTIONS</strong> <strong>TO</strong> <strong>PRACTICE</strong> PROBLEMS 3<br />

(b) Is S a basis for R 3 ?<br />

SOLUTION: No, S is not a basis for R 3 . Any basis must be linearly<br />

independent (see Theorem 2.1 in the textbook), and by part (a), S is not<br />

linearly independent.<br />

(c) Find a basis for span(S).<br />

SOLUTION: To find the basis, we put the vectors into a matrix A as<br />

columns and row reduce to row echelon form, as in part (a). Since there are<br />

pivots in columns 1 and 2 (but not in column 3), a basis for the span of S is<br />

the first two vectors in S:<br />

⎧⎡<br />

⎤ ⎡ ⎤⎫<br />

⎨<br />

⎣ 1 2 ⎦ ,<br />

⎩<br />

3<br />

⎣ 4 5<br />

6<br />

⎬<br />

⎦<br />

⎭ .<br />

Problem 4. For each of the following matrices, determine whether or not<br />

it is invertible. If it is invertible, find its inverse.<br />

⎡<br />

[ ]<br />

2 1<br />

A =<br />

B = ⎣ 1 0 1<br />

⎤<br />

[ ]<br />

0 1 0 ⎦ 1 0 1<br />

C =<br />

1 2<br />

0 1 0<br />

1 0 1<br />

Solution:<br />

D =<br />

⎡<br />

⎢<br />

⎣<br />

0 1 0 0<br />

1 0 0 0<br />

2 0 1 0<br />

2<br />

3 0 0 −1<br />

⎤<br />

⎥<br />

⎦ .<br />

The matrix A is invertible:<br />

[ ] [ ] [<br />

2 1 | 1 0 2 1 | 1 0 1<br />

1 1<br />

| 0<br />

∼<br />

1 2 | 0 1 0 3 | − 1 ∼ 2 2<br />

1 0 3 | − 1 1<br />

2 2 2 2<br />

[ ] [ ]<br />

1<br />

1 1<br />

| 0<br />

∼ 2 2 1 0 |<br />

2<br />

−<br />

0 1 | − 1 2 ∼<br />

1 3 3<br />

0 1 | − 1 2 .<br />

3 3<br />

3 3<br />

]<br />

The matrix B is not invertible. We can see this is by counting its pivots:<br />

by subtracting Row 1 from Row 3, we get the reduced row echelon matrix<br />

⎡<br />

⎣ 1 0 1<br />

⎤<br />

0 1 0 ⎦ ,<br />

0 0 0


4 JOHN GOODRICK<br />

which has pivots in only 2 of the 3 columns, and so B cannot be row reduced<br />

to I 3 . Therefore B is not invertible.<br />

The matrix C is not invertible, simply because it is not a square matrix.<br />

Recall that in the definition of an invertible matrix (on page 75 of the textbook),<br />

we require that the matrix be n × n for some n.<br />

For the matrix D, we augment the matrix by I 4 on the right and then rowreduce<br />

the left half (as in section 1.5):<br />

∼<br />

⎡<br />

⎢<br />

⎣<br />

⎡<br />

⎢<br />

⎣<br />

0 1 0 0 | 1 0 0 0<br />

1 0 0 0 | 0 1 0 0<br />

2 0 1 0 | 0 0 1 0<br />

2<br />

3 0 0 −1 | 0 0 0 1<br />

1 0 0 0 | 0 1 0 0<br />

0 1 0 0 | 1 0 0 0<br />

0 0 1 0 | 0 −2 1 0<br />

2<br />

0 0 0 −1 | 0 −3 0 1<br />

⎤<br />

⎡<br />

⎥<br />

⎦ ∼ ⎢<br />

⎣<br />

⎤ ⎡<br />

⎥<br />

⎦ ∼ ⎢<br />

⎣<br />

1 0 0 0 | 0 1 0 0<br />

0 1 0 0 | 1 0 0 0<br />

2 0 1 0 | 0 0 1 0<br />

2<br />

3 0 0 −1 | 0 0 0 1<br />

⎤<br />

⎥<br />

⎦<br />

1 0 0 0 | 0 1 0 0<br />

0 1 0 0 | 1 0 0 0<br />

0 0 1 0 | 0 −4 2 0<br />

0 0 0 1 | 0 3 0 −1<br />

Since we were able to row-reduce the left half to the identity matrix I 4 , the<br />

matrix D is invertible, and the row reduction above shows that<br />

⎡<br />

⎤<br />

0 1 0 0<br />

D −1 = ⎢ 1 0 0 0<br />

⎥<br />

⎣ 0 −4 2 0 ⎦ .<br />

0 3 0 −1<br />

Problem 5. (a) Write the solution set of<br />

3x 1 + 2x 2 + 7x 3 + x 4 = 2<br />

3x 1 + 3x 2 + 6x 3 + x 4 = 17<br />

as ⃗p + span(⃗v 1 , ⃗v 2 ) for some vectors ⃗p, ⃗v 1 , and ⃗v 2 .<br />

SOLUTION: First, you should find the general solution by the method in<br />

section 1.4:<br />

[<br />

3 2 7 1 | 2<br />

3 3 6 1 | 17<br />

⎤<br />

⎥<br />

⎦ .<br />

] [ ] [ ]<br />

3 2 7 1 | 2 1<br />

2 7 1 2<br />

|<br />

∼<br />

∼ 3 3 3 3<br />

0 1 −1 0 | 15 0 1 −1 0 | 15<br />

[ ]<br />

1 0 3<br />

1<br />

| −<br />

∼<br />

28 3 3 .<br />

0 1 −1 0 | 15<br />

This is in reduced row echelon form, so we can read of the general soution:<br />

x 1 = −3x 3 − 1 3 x 4 − 28 3 , x 2 = x 3 + 15, and x 3 and x 4 are free.<br />

From here, we can proceed as in section 1.6: the solution set consists of all<br />

(x 1 , x 2 , x 3 , x 4 ) in R 4 such that


<strong>MATH</strong> <strong>1105</strong>, <strong>PARTIAL</strong> <strong>EXAM</strong> 1 <strong>SOLUTIONS</strong> <strong>TO</strong> <strong>PRACTICE</strong> PROBLEMS 5<br />

⎡ ⎤ ⎡<br />

x 1<br />

⎢ x 2<br />

⎥<br />

⎣ x 3<br />

⎦ = ⎢<br />

⎣<br />

x 4<br />

−3x 3 − 1 3 x 4 − 28 3<br />

x 3 + 15<br />

x 3<br />

x 4<br />

⎤<br />

⎥<br />

⎦ = x 3<br />

⎡<br />

⎢<br />

⎣<br />

−3<br />

1<br />

1<br />

0<br />

⎤<br />

⎥<br />

⎦ + x 4<br />

⎡<br />

⎢<br />

⎣<br />

− 1 3<br />

0<br />

0<br />

1<br />

⎤ ⎡<br />

⎥<br />

⎦ + ⎢<br />

⎣<br />

Therefore we can let ⃗p = [ − 28 3 , 15, 0, 0] , ⃗v 1 = [−3, 1, 1, 0], and ⃗v 2 = [ − 1 3 , 0, 0, 1] .<br />

(This is not the only possible solution to this problem.)<br />

(b) Write the solution set of<br />

as the span of two vectors.<br />

3x 1 + 2x 2 + 7x 3 + x 4 = 0<br />

3x 1 + 3x 2 + 6x 3 + x 4 = 0<br />

SOLUTION: Note that this homogeneous linear system is the same as the<br />

system in part (a) except that the numbers on the right sides of the equations<br />

are 0. So using the same row reduction as in part (a), we get:<br />

[<br />

3 2 7 1 | 0<br />

3 3 6 1 | 0<br />

]<br />

∼<br />

[<br />

1 0 3<br />

1<br />

3<br />

| 0<br />

0 1 −1 0 | 0<br />

Now using the same technique as in part (a), we can see that the general<br />

solution is<br />

⎡ ⎤ ⎡<br />

x 1 −3x 3 − 1 ⎢ x 2<br />

⎥<br />

⎣ x 3<br />

⎦ = x ⎤ ⎡<br />

3 4<br />

⎢ x 3<br />

⎥<br />

⎣ x 3<br />

⎦ = x ⎢<br />

3 ⎣<br />

x 4 x 4<br />

with x 3 and x 4 free. So the solution set is<br />

⎛⎡<br />

⎤ ⎡<br />

−3 − 1 3<br />

span ⎜⎢<br />

1<br />

⎥<br />

⎝⎣<br />

1 ⎦ , ⎢ 0<br />

⎣ 0<br />

0 1<br />

−3<br />

1<br />

1<br />

0<br />

⎤<br />

⎥<br />

⎦ + x 4<br />

⎤⎞<br />

⎥⎟<br />

⎦⎠ .<br />

Problem 6. Let S be the set of all vectors ⃗v in R 3 such that<br />

⃗v · [1, 3, 11] = 0.<br />

Is S a subspace of R 3 ? Why or why not?<br />

SOLUTION: Yes, the set S is a subspace of R 3 . One way to see this is to<br />

note that S is the set of all ⃗v = [v 1 , v 2 , v 3 ] such that<br />

v 1 + 3v 2 + 11v 3 = 0,<br />

⎡<br />

⎢<br />

⎣<br />

]<br />

.<br />

− 1 3<br />

0<br />

0<br />

1<br />

⎤<br />

⎥<br />

⎦ ,<br />

− 28 3<br />

15<br />

0<br />

0<br />

⎤<br />

⎥<br />

⎦ .


6 JOHN GOODRICK<br />

which is a homogeneous linear system (with only one equation), and as explained<br />

in section 1.6, the solution set to a homogeneous linear system is always<br />

a subspace of the appropriate R n .<br />

Alternatively, you could have checked that the zero vector [0, 0, 0] is in S<br />

and that S is closed under vector addition and scalar multiplication – see<br />

Definition 1.16 on page 89 of the textbook.<br />

Problem 7. For how many different vectors ⃗v in R 3 is it true that<br />

⎧⎡<br />

⎨<br />

Span ⎣ 1 ⎤ ⎡<br />

0 ⎦ , ⎣ 0 ⎤ ⎫<br />

⎬<br />

1 ⎦ , ⃗v<br />

⎩<br />

0 0<br />

⎭ = R3 ?<br />

Explain.<br />

SOLUTION: There are infinitely many such vectors ⃗v.<br />

Here one way to prove this: if we write<br />

⎡<br />

⃗v = ⎣ v ⎤<br />

1<br />

v 2<br />

⎦ ,<br />

v 3<br />

then by Theorem 1.16 of section 1.6, the span of the set<br />

⎧⎡<br />

⎨<br />

⎣ 1 ⎤ ⎡<br />

0 ⎦ , ⎣ 0 ⎤ ⎫<br />

⎬<br />

1 ⎦ , ⃗v<br />

⎩<br />

0 0<br />

⎭<br />

is all of R 3 if and only if the matrix<br />

⎡<br />

A =<br />

⎣ 1 0 v ⎤<br />

1<br />

0 1 v 2<br />

⎦<br />

0 0 v 3<br />

is row equivalent to I 3 .<br />

The matrix A is already in row echelon form. It has either two or three<br />

pivots, depending on whether or not v 3 = 0: the two 1’s are pivots in any case,<br />

the v 3 in the lower right is a pivot if it is nonzero, and otherwise v 3 is not a<br />

pivot. As long as v 3 ≠ 0, we can row reduce the matrix A to I 3 by adding<br />

appropriate multiples of row 3 to rows 1 and 2. (The values of v 1 and v 2 do<br />

not matter.)<br />

Since there are infinitely many such choices for v 3 (it can be any nonzero<br />

number in R), there are infinitely many such vectors ⃗v.<br />

Problem 8. Is the vector (0, 3, 1) in<br />

⎧⎡<br />

⎨<br />

Span ⎣ 1 ⎤ ⎡<br />

2 ⎦ , ⎣ 2 ⎤⎫<br />

⎬<br />

1 ⎦<br />

⎩<br />

1 1<br />

⎭ ?<br />

Explain why or why not.


<strong>MATH</strong> <strong>1105</strong>, <strong>PARTIAL</strong> <strong>EXAM</strong> 1 <strong>SOLUTIONS</strong> <strong>TO</strong> <strong>PRACTICE</strong> PROBLEMS 7<br />

SOLUTION: Yes, the vector (0, 3, 1) is in this span.<br />

The vector (0, 3, 1) is in this span if and only if the system<br />

⎡ ⎤ ⎡ ⎤ ⎡ ⎤<br />

x 1 ·<br />

⎣ 1 2<br />

1<br />

⎦ + x 2 ·<br />

⎣ 2 1<br />

1<br />

⎦ =<br />

⎣ 0 3<br />

1<br />

has a solution for x 1 and x 2 . But this is equivalent to the linear system<br />

x 1 + 2x 2 = 0<br />

2x 1 + x 2 = 3<br />

x 1 + x 2 = 1.<br />

We convert this into an augmented matrix, then row reduce:<br />

⎡<br />

⎣ 1 2 | 0<br />

2 1 | 3<br />

1 1 | 1<br />

⎤<br />

⎦ ∼<br />

⎡<br />

⎣ 1 2 | 0<br />

0 −3 | 3<br />

0 −1 | 1<br />

⎤<br />

⎦ ∼<br />

⎡<br />

⎦<br />

⎣ 1 2 | 0<br />

0 −3 | 3<br />

0 0 | 0<br />

This last matrix is in row echelon form, with pivots in columns 1 and 2, but<br />

no pivot in the rightmost column. Therefore the system is consistent (it has a<br />

solution).<br />

⎤<br />

⎦ .

Hooray! Your file is uploaded and ready to be published.

Saved successfully!

Ooh no, something went wrong!